LSAT and Law School Admissions Forum

Get expert LSAT preparation and law school admissions advice from PowerScore Test Preparation.

User avatar
 Dave Killoran
PowerScore Staff
  • PowerScore Staff
  • Posts: 5853
  • Joined: Mar 25, 2011
|
#42037
Complete Question Explanation
(The complete setup for this game can be found here: lsat/viewtopic.php?t=15987)

The correct answer choice is (D)

Answer choice (A) is incorrect because H cannot be placed first.

Answer choice (B) is incorrect because Z must immediately precede M.

Answer choice (C) is incorrect because H cannot immediately precede O.

Answer choice (D) is the correct answer.

Answer choice (E) is incorrect because X and V must be separated by the same number of files as separate H and O.

Get the most out of your LSAT Prep Plus subscription.

Analyze and track your performance with our Testing and Analytics Package.